Что такое фотон?

Рассмотрим вопрос: «Что такое фотон?» . Ответы говорят: «элементарная частица» и не более того. На самом деле они не отвечают на вопрос. Более того, вопрос помечен как дубликат вопроса «Что такое квант света?» – ответы там тоже не говорят мне, что такое фотон. Ни один из ответов на этот вопрос не упоминается в комментариях. Когда я ищу "фотон" , я не могу найти ничего полезного. Такие вопросы, как «Волновая функция фотона» , выглядят многообещающе, но не приносят плодов. Другие говорят что-то вроде «фотон — это возбуждение фотонного поля». Это ничего мне не говорит. И неописание тега , в котором говорится:

Фотон является квантом электромагнитного четырехпотенциала и, следовательно, безмассовой бозонной частицей, связанной с электромагнитной силой, обычно также называемой «частицей света»...

Я бы сказал, что это менее чем полезно, потому что создается впечатление, что фотоны вечно возникают и летают туда-сюда, проявляя силу. Эта же концепция есть и в статье о фотонах в Википедии , но это неправда. Как сказала Анна , «виртуальные частицы существуют только в математике модели». Итак, кто может сказать мне, что такое реальный фотон, или порекомендовать мне какое-то авторитетное информативное определение, которое принято и которому доверяют физики элементарных частиц? Я говорю все это, потому что считаю, что это имеет первостепенное значение. Если у нас нет ясного представления о том, что такое фотон, у нас нет основания. Это как то, что сказал котозна :

Фотоны кажутся одной из основополагающих идей квантовой механики, поэтому меня беспокоит, что без четкого определения или набора конкретных примеров основа для понимания квантовых экспериментов будет несколько размытой.

Я второй, только больше. Как мы можем понять образование пар, если мы не понимаем, что такое фотон? Или электрон? Или электромагнитное поле? Или все остальное? Все начинается с фотона.

Я дам награду в 400 баллов за наименее худший ответ на вопрос. Один ответ получит награду, даже если мне это не нравится. А вопрос вот в чем:

Что такое фотон?

Люди могут найти эту статью полезной; Существуют ли фотоны на самом деле? С.А. Рашковский
Интересно! Автор утверждает, что многие вещи, для которых обычно требуются дискретные фотоны, можно объяснить с помощью более или менее непрерывных волн излучения и дискретных атомов, зарядов и т. д. Знаете ли вы, остались ли какие-нибудь вещи, для которых действительно требуются дискретные кванты фотонов? свет, помимо тех, на которые он смотрит?
Это легко, Джон. Как я уже говорил вам в нашем предыдущем разговоре, фотон — это просто двумерная замкнутая петля (круг) электромагнитного потока, движущегося в пространстве с c. Более или менее похоже на вихревое кольцо. Периметр 2πr петли — это длина волны фотона. Спин круга равен 1.
Возможно, настоящий фотон — это энергетические пакеты, которые мы поглощаем по пути.
На этой странице отсутствует слово «гравитация». Отклоняется ли луч света вблизи массы для независимого наблюдателя? Луч света состоит из фотонов? Изменился ли импульс фотонов вблизи массы для независимого наблюдателя? Изменился ли импульс массы в соответствии с законом сохранения импульса для независимого наблюдателя? Возможно ли это, если у фотона нет определенной траектории между испусканием и поглощением?
«... фотон - это энергетические пакеты ...» Вы имеете в виду, как глюон, это то, как массы обмениваются массами (с - константа, а направление искривлено: на мой взгляд, обмен было бы неправильно сказано, так как массы выталкивают фотоны , другие массы их подхватывают) извините за спонтанность.

Ответы (21)

Слово фотон — одно из самых запутанных и неправильно используемых слов в физике. Вероятно, гораздо больше, чем другие слова в физике, оно используется в нескольких различных значениях, и можно только попытаться определить, какое из них имеется в виду, исходя из источника и контекста сообщения.

Фотон, который экспериментатор по спектроскопии использует для объяснения того, как спектры связаны с атомами и молекулами , отличается от понятия фотона, о котором говорят экспериментаторы по квантовой оптике, объясняя свои эксперименты. Они отличаются от фотона, о котором говорят экспериментаторы высоких энергий, и есть еще другие фотоны, о которых говорят теоретики высоких энергий. Вероятно, существует еще больше вариантов (и бесчисленное количество личных модификаций).

Термин был введен Г. Н. Льюисом в 1926 г. для понятия «атом света»:

[...] может возникнуть соблазн принять гипотезу о том, что мы имеем дело с новым типом атома, идентифицируемой сущностью, несотворимой и неразрушимой, которая действует как носитель лучистой энергии и после поглощения сохраняется как существенная составляющая поглощающего атома, пока он позже не будет отправлен снова с новым количеством энергии [...]
- «Происхождение слова« фотон »»

Поэтому я беру на себя смелость предложить для этого гипотетического нового атома, который не является светом, но играет существенную роль в каждом процессе излучения, имя фотон.
- «Сохранение фотонов» (1926-12-18).

Насколько я знаю, это первоначальное значение слова фотон больше не используется, потому что все современные варианты допускают создание и уничтожение фотонов.

Фотон, о котором обычно говорит экспериментатор в видимой и ультрафиолетовой спектроскопии, — это объект с определенной частотой. ν и определенная энергия час ν ; его размер и положение неизвестны, возможно, не определены; тем не менее, он может быть поглощен и испущен молекулой.

Фотон, о котором обычно говорит экспериментатор в области квантовой оптики (детектирующие корреляционные исследования), представляет собой намеренно таинственный «квантовый объект», который более сложен: он не имеет определенной частоты, имеет несколько определенное положение и размер, но может охватывать всю экспериментальную установку и только выглядит как локализованная частица, когда ее обнаруживают в детекторе света.

Фотон, о котором говорит экспериментатор высоких энергий, — это маленькая частица, которую невозможно увидеть на фотографиях треков частиц и событий их рассеяния, но которая позволяет легко объяснить кривизну треков частиц материи с общей точкой происхождения в рамках сохранения энергии и импульса (например, появление пары противоположно заряженных частиц или комптоновское рассеяние). Этот фотон обычно имеет определенный импульс и энергию (следовательно, также определенную частоту) и довольно определенное положение, поскольку он участвует в достаточно локализованных актах рассеяния.

Теоретики также используют слово фотон в нескольких значениях. Общим знаменателем является математика, используемая для описания электромагнитного поля и его взаимодействия с материей. Определенные особые квантовые состояния электромагнитного поля - так называемые состояния Фока - математически ведут себя таким образом, что это позволяет использовать язык "фотонов как исчисляемых вещей с определенной энергией". Точнее, существуют состояния электромагнитного поля, которые можно указать, задав бесконечный набор неотрицательных целых чисел. Когда одно из этих чисел меняется на единицу, это описывается фигурой речи как «сотворение фотона» или «разрушение фотона». Такой способ описания состояния позволяет легко вычислить полную энергию системы и ее частотное распределение. Однако этот тип фотона не может быть локализован, кроме как во всей системе.

В общем случае состояние ЭМ поля не является таким уж особым, и само число фотонов не является определенным. Это означает, что основным объектом математической теории ЭМ поля является не множество точечных частиц с определенным числом членов, а непрерывное ЭМ поле. Фотоны — это просто фигура речи, полезная, когда поле особого рода.

Тем не менее, теоретики все еще много говорят о фотонах, отчасти потому, что:

  • он прочно укоренился в программе и учебниках по историческим и инерционным причинам;

  • экспериментаторы используют его для описания своих экспериментов;

  • отчасти потому, что она производит хорошее впечатление на людей, читающих популярные книги по физике; трудно интересно говорить о ψ функция или фоковское пространство, но легко говорить о «частицах света»;

  • частично из-за того, как преподается метод диаграмм Фейнмана .

(На диаграмме Фейнмана волнистая линия в пространстве-времени часто изображается фотоном. Но эти диаграммы помогают в расчетах теории возмущений для сложных уравнений поля; волнистая линия на диаграмме Фейнмана не обязательно представляет реальную точечную частицу, движущуюся через Диаграмма вместе с фотоном, к которому она относится, является просто полезным графическим представлением некоторых сложных интегралов.)

Замечание о необходимости концепции фотона

Многие известные эксперименты, когда-то считавшиеся свидетельством существования фотонов, позже были объяснены качественно или полуколичественно, основываясь исключительно на теории волн (классическая электромагнитная теория света, иногда с добавлением уравнения Шредингера). Это, например, фотоэлектрический эффект , комптоновское рассеяние , излучение черного тела и, возможно, другие.

Всегда существовало меньшинство физиков, которые вообще избегали понятия фотона для такого рода явлений и предпочитали идею о том, что возможности ЭМ теории не исчерпаны. Ознакомьтесь с этими статьями о нефотонных подходах к физике:

Р. Кидд, Дж. Ардини, А. Антон, Эволюция современного фотона, Am. Дж. Физ. 57, 27 (1989) http://www.optica.machorro.net/Lecturas/ModernPhoton_AJP000027.pdf

CV Raman, Классический вывод эффекта Комптона. Индийский журнал физики, 3, 357–369. (1928) http://dspace.rri.res.in/jspui/bitstream/2289/2125/1/1928%20IJP%20V3%20p357-369.pdf

Тревор В. Маршалл, Эмилио Сантос: Миф о фотоне, Arxiv (1997) https://arxiv.org/abs/quant-ph/9711046v1

Тимоти Х. Бойер, Вывод спектра излучения абсолютно черного тела без квантовых предположений, Phys. 182, 1374 (1969) https://dx.doi.org/10.1103/PhysRev.182.1374

Вероятно, стоит дать ссылку на статью/мнение Лэмба «Антифотон» . К сожалению, у меня нет ссылки без платного доступа.
@dmckee: спасибо. Я нашел бумагу . Потому что вы упомянули об этом пару лет назад!
Короче говоря, фотон — это не совсем так.
@JohnDuffield Вам должен нравиться непосредственно их комментарий , а не сам вопрос;)
До Льюиса существовало другое определение фотона: одна свеча на квадратный метр на площади в один квадратный миллиметр. См. стр. 173, сноску 20, в статье 1920 года здесь: books.google.com/… , устройство теперь называется троландом en.wikipedia.org/wiki/Troland .
+1, за «он используется в нескольких разных значениях, и можно только попытаться определить, какое из них имеется в виду, исходя из источника и контекста сообщения». многозначность всегда является источником путаницы. Более того, работа, необходимая для того, чтобы привести объяснителя к этой, POV, реализации, отправной точке, может быть непреодолимой.
Это очень странное определение того, как высокоэнергетические экспериментаторы относятся к фотонам. Реальные фотоны — это жесткие гамма-лучи, наблюдаемые с помощью электромагнитных потоков в калориметрах. Они также производятся в сцинтилляционных детекторах и через тормозное излучение (ср. Кривизна трека). Мы также много говорим о них как об интегральных пропагаторах Фейнмана и, вероятно, немного бесцеремоннее, чем следовало бы, называть их «частицами» в этом контексте ;-)
@andybuckley, я только упомянул то, что, по моему мнению, является отличительной чертой концепции экспериментатора HEP, стоящей за словом фотон - частица, которую нельзя наблюдать напрямую, но которая имеет положение, энергию и импульс, что помогает объяснить взаимодействие заряженных частиц между собой и их поведение во внешнем ЭМ поле (эффект Комптона, создание пар). Конечно, есть и другие явления, подобные тем, которые вы упомянули. Было бы здорово, если бы вы могли подробнее описать, что представляет собой экспериментаторская концепция фотона.
Мне очень нравится ответ, мне действительно не нравится вопрос. Вопрос кажется пронизанным научной разглашающей литературой, интерпретируемой как настоящее исследование физики. Это по-прежнему очень актуальный вопрос, но кажется, что правильный ответ — «ну, окопаться в поле». Я ученый в области сверхбыстрой / квантовой оптики, и я регулярно сталкиваюсь с тремя из приведенных выше «определений». Для меня нет двусмысленности. Я чувствую, что после того, как концепции хорошо закреплены, все они хорошо взаимодействуют друг с другом. (1/2)
(2/2) Это как есть лук. Попробовав его, вы узнаете, было ли оно вареным, жареным, обжаренным, жареным или сырым. Но это все же лук. Если я читаю статью, необходимые понятия легко вспоминаются. Сказать фотон для всех из них не дает мне двусмысленности. Свет — странная вещь, которая во многом зависит от того, как он создается, от его силы, от его взаимодействия с самим собой и с материей. Но все это фотоны, электромагнитные поля.
(конечно, вы можете возразить: «но у вас также есть термин «вареный лук» или «жареный лук», которые затем относятся к конкретной вещи». Да, но тогда становится сложнее отслеживать вещи. Узнав, что такое Фотон и ароматы, в которых он проявляется, вы расширяете свое понимание и видите сходство между различными областями.Шеф-повар должен уметь готовить лук всеми способами, а не только одним, даже если он специализировался на одном конкретном виде!)

Фотон — это конструкция, которая была введена для объяснения экспериментальных наблюдений, показавших, что электромагнитное поле поглощается и излучается квантами. Многие физики принимают эту конструкцию за указание на то, что электромагнитное поле состоит из безразмерных точечных частиц, однако в этом конкретном факте нельзя быть абсолютно уверенным. Все экспериментальные наблюдения, связанные с электромагнитным полем, обязательно связаны с процессом поглощения и/или излучения.

Итак, когда дело доходит до строго онтологического ответа на вопрос «Что такое фотон?» мы должны быть честными и сказать, что мы действительно не знаем. Это похоже на те старые вопросы о сущности вещей; вопрос, на который никогда нельзя было дать удовлетворительного ответа. Путь к лучшему пониманию часто требует, чтобы человек привык к неопределенности.

Комментарии не для расширенного обсуждения; этот разговор был перемещен в чат .
Это довольно поверхностный ответ. У слова «фотон» в квантовой теории поля есть технические значения. DavidZ намекает на это здесь , и я немного поясню понятие частицы, например , здесь . Кроме того, его можно найти в большинстве стандартных методов лечения КЭД. Ваши утверждения об онтологии верны, но на самом деле это не физический ответ на вопрос. То, что спрашивающий кажется неудовлетворенным или незнакомым с этими техническими значениями, не означает, что мы не должны их объяснять.
@ACuriousMind, да, это правда, что у нас есть более техническая трактовка фотона в квантовой теории поля и т. д. Однако мне показалось, что человек, задавший вопрос, знает об этих трактовках и недоволен ими. Вопрос, кажется, требует онтологического ответа.
Разве квант света не был «изобретен» для помощи в квантовой механике? Или было наоборот...
«Фотон — это конструкция, введенная для объяснения экспериментальных наблюдений». Это можно сказать о любом продукте человеческого интеллекта. Вопрос в том, представляет ли эта конструкция частицу — что бы это ни значило — или удобную фикцию.
Как электроны обмениваются массой?

Это таблица элементарных частиц, используемая в стандартной модели физики элементарных частиц , которая постоянно проверяется на БАК , несмотря на обнадеживающие поиски расширений.

Таблица частиц

Стандартная модель элементарных частиц (более схематичное изображение) с тремя поколениями материи, калибровочными бозонами в четвертом столбце и бозоном Хиггса в пятом.

Обратите внимание на слово «частица» и обратите внимание, что всякий раз, когда физический процесс рассчитывается так, чтобы дать числа для сравнения с экспериментальными измерениями, эти частицы рассматриваются как точечные частицы. то есть на этих диаграммах Фейнмана для рассеяния фотонов:

комп

Прилетающий фотон реален, т.е. на массовой оболочке, масса 0, энергия час ν . Вершина — это точка, и именно поэтому физики, занимающиеся элементарными частицами, продолжают говорить о точечных частицах (до тех пор, пока, возможно, не будет подтверждена теория струн, и тогда мы будем говорить о струнных частицах). Представление о фотоне как о частице столь же реалистично, как и представление об электроне, и его существование подтверждается соответствием данным предсказаний стандартной модели.

Таким образом, ответ заключается в том, что фотон — это частица в стандартной модели физики, которая соответствует измерениям в квантово-механических измерениях, то есть измерениях, соизмеримых с .

Всматриваться в «значение фотона» больше, чем в «значение электрона» в математической модели, — это уже не физика, а метафизика. т.е. люди переносят свои убеждения предрассудков на объяснение.

Мы называем электрон «частицей» в наших экспериментальных установках, потому что макроскопический след, когда он проходит через детекторы, такой же, как у классической частицы. То же самое верно и для фотонов, измеренных в калориметрах БАК, их макроскопический «след» представляет собой частицу с нулевой массой с энергией час ν и крутить один.

ЦМС дифотон даже

Это дифотонное событие CMS не вызывает сомнений, является ли след фотоном или нет. Это фотон таблицы элементарных частиц. Только в вершинах взаимодействия важна квантово-механическая неопределенность.

Ты спрашиваешь:

Как мы можем понять образование пар, если мы не понимаем, что такое фотон?

Кажется, что нужно постоянно подчеркивать, что физические теории моделируют данные ; они не являются метафизическим предположением о том, как возник мир. У нас есть успешная модель QFT для физики элементарных частиц, которая описывает поведение элементарных частиц по мере того, как их след фиксируется в экспериментах, и успешно предсказывает новые результаты. Это все.

Мы понимаем процессы, моделируемые КТП , понимание природы аксиоматической установки частиц в таблице принадлежит метафизике. Принимая постулаты квантовой механики и принимая частицы в таблице, мы можем моделировать взаимодействия частиц. Это похоже на вопрос «почему С U ( 3 ) × С U ( 2 ) × U ( 1 ) «Единственный ответ заключается в том, что модель, основанная на этих предположениях, описывает существующие данные о частицах и успешно предсказывает новые установки.

Я хотел бы дать ссылку на сообщение в блоге Motl , которое помогает понять, как классическое электромагнитное поле возникает в результате большого слияния фотонов. Для этого нужна математика квантовой теории поля. Электрические и магнитные поля присутствуют в волновой функции фотона, которая является функцией комплексного числа и не поддается измерению, за исключением ее комплексно-сопряженного квадрата, действительное число дает плотность вероятности нахождения фотона в ( Икс , у , г , т ) .

Это суперпозиция бесчисленных волновых функций фотонов, которая создает классическую электромагнитную волну. Частота волновой функции отдельного фотона проявляется в комплексных показателях, описывающих ее. Неудивительно, что это та же частота вероятности, что и частота классической электромагнитной волны, возникающей из бесчисленных фотонов той же энергии (той же частоты). Оба математических выражения основаны на структуре уравнений Максвелла, фотон — в квантованной форме, ЭМ — в классических уравнениях.

Да и не зря это называют физикой элементарных частиц.
Могу ли я получить некоторое представление о том, что можно увидеть на картинке? Это кажется слишком сложным для столкновения двух фотонов, даже если мы посмотрим только на желтые следы.
@JanDvorak, если вы посмотрите на ссылку над картинкой, это протонное рассеяние протонов в детекторе CMS, а две синие гистограммы - это два фотона высокой энергии, возникающие в результате взаимодействия, измеренного в электромагнитных калориметрах. Остальные — заряженные частицы разного типа, и нужен конкретный анализ события, чтобы увидеть, есть ли там что-то более интересное.
Анна, у меня есть вопрос специально к вам, потому что я думаю, что вы, возможно, одна из очень немногих людей, которые могут на него ответить. Это относится к рассеянному изображению LHC. Если предположить, что изображенное столкновение может повториться ИДЕНТИЧНО, как оно произошло, будут ли два фотона рассеиваться точно так же? Снова предположим, что столкновение могло бы повториться абсолютно идентично.
@Inquisitive Точно так же можно управлять только входящим рассеянием протона на протон, то есть той же энергией / импульсом. Это и есть эксперимент LHC. Идентичные входные рассеяния и измерение исходящих созданных частиц. Таким образом, эксперимент говорит, что каждое событие отличается, даже если входные данные идентичны. Исследование зависит от стандартной модели физики элементарных частиц en.wikipedia.org/wiki/Standard_Model , которая позволяет изучать категории, например cds.cern.ch/record/1378102 . В квантово-механическом режиме выход является вероятностным
Стандартная модель предсказывает распределения вероятностей для разбросов с использованием входных данных из предыдущих экспериментов, и на данный момент она очень хорошо проверена.
Спасибо, Анна. Еще один. Из вашего ответа могу ли я сделать следующий вывод? Если бы столкновение произошло снова и абсолютно идентично первому столкновению, два фотона действительно могли бы разлететься в разные стороны от первого столкновения.
@ Любознательный . Да. есть только вероятность для каждого события, поддающаяся вычислению, с помощью SM One получают распределения и сравнивают их с предсказанными распределениями.
Спасибо за ваше время Анна. Для меня это невероятно, если я правильно об этом думаю.
@Inquisitive Это была причина, по которой квантовая механика как теория должна была быть изобретена. Все данные микромира противоречат нашим интуитивным классическим представлениям о мире.
Анна, я был склонен предполагать (из-за поверхностного знакомства с теорией Эйнштейна-Картана), что электрон может иметь пространственную протяженность (на два или три порядка больше планковской длины), требуемую от фермионов в этой теории, так что, хотя это немного не по теме в отношении рассматриваемого вопроса 2016 года, кажется ли вам правдоподобным, что «след» электрона в вершинах может представлять только центр его массы? (Я надеюсь получить ваш ответ, чтобы, имея под рукой схему и диаграммы, я мог исключить эту возможность из своих предположений.)
@Edouard Что касается экспериментов, то след электрона в детекторах ограничен экспериментальными ошибками. Это стандартная модель физики элементарных частиц, в которой все элементарные частицы являются точечными частицами в квантовой теории поля. Поскольку СМ очень успешно отображает существующие данные и предсказывает новые, существующая теория частиц рассматривает электрон и остальные элементарные частицы как точечные частицы. Любая другая теория должна была бы включать стандартную модель, чтобы иметь шанс быть более правильной. Экспериментальный предел радиуса электрона намного больше
( en.wikipedia.org/wiki/Electron ), чем планковская длина, поэтому поле открыто, пока окончательная теория соответствует уже существующим данным, что будет, если она встроит стандартную модель.
@Edouard Примером может служить теория струн, которая имеет симметрии, которые могут включать стандартную модель, и предполагает, что частицы являются возбуждениями струны, а не точкой. (Теоретикам так и не удалось найти ни одного из тысяч возможных.)

Отправной точкой для объяснения фотонов с теоретической точки зрения должны быть уравнения Максвелла. В ковариантной форме уравнения в вакууме без источников имеют вид

мю Ф мю ν знак равно 0 мю ( ϵ мю ν α β Ф α β ) знак равно 0
Хорошо известно, что второе уравнение автоматически проверяется, если Ф определяется с точки зрения потенциала А
Ф мю ν знак равно мю А ν ν А мю
Уравнения Максвелла могут быть получены из лагранжиана
л знак равно 1 4 Ф мю ν Ф мю ν
когда используются Эйлера-Лагранжа, изменяющие потенциал .

Этот классический лагранжиан является основой для формулировки квантовой теории поля. Поскольку уравнения Максвелла определяют классическую теорию поля, естественно искать описание КТП, а не только описание КМ. Не вступая в обсуждение смысла квантования (что было бы слишком математически-философским и не освещало бы ваш вопрос), давайте предположим, что формулировка КТП может быть выполнена, эквивалентно, через интеграл по путям и каноническое квантование. Я буду обсуждать только последнее.

При каноническом квантовании потенциал А мю и его сопряженный импульс Π мю знак равно л ( 0 А мю ) становятся полевыми операторами, действующими в некотором гильбертовом пространстве. Эти операторы вынуждены удовлетворять коммутационному соотношению

[ А мю ( т , Икс ) , Π ν ( т , Икс ) ] знак равно я η мю ν дельта ( Икс Икс )

Благодаря этому соотношению две физические поляризации для А могут быть расширены в нормальных режимах, которые следует интерпретировать как операторы уничтожения и создания, а а также а . Если вакуумное состояние (т. е. состояние минимальной энергии теории) | 0 , то состояния а | 0 называются однофотонными состояниями. Поэтому фотон есть минимальное возбуждение квантового электромагнитного потенциала .

Все вышесказанное касается только свободных электромагнитных полей. Это означает, что фотоны распространяются вечно, они не могут испускаться или поглощаться. Это явно противоречит реальной жизни (и слишком скучно).

Возвращаясь к классическому электромагнетизму, лагранжиан для ЭМ поля с 4-токовым Дж который действует как источник

л знак равно 1 4 Ф мю ν Ф мю ν А мю Дж мю
Самый распространенный пример — ток, создаваемый заряженным фермионом (например, электроном или мюоном)
Дж мю знак равно я е ψ ¯ мю ψ

Но у этого типа лагранжианов есть огромный недостаток: мы не знаем, как их точно проквантовать . * Вещи становятся беспорядочными с взаимодействиями. Мы можем говорить лишь с некоторой строгостью об асимптотических состояниях: состояния задолго до или задолго после любого взаимодействия напоминают состояния свободных полей. Поэтому реальный фотон есть возбуждение квантового электромагнитного потенциала, который в пределе т ± стремится к свободному фотону , как определено выше.

Так что да, в некотором смысле вы правы, что мы не знаем, что такое фотон. Но это [формальное] препятствие не мешает нам делать предсказания, как в случае с рождением пар, который вас беспокоит. Ключевым моментом является то, что мы не знаем, что происходит во время взаимодействия, мы не можем этого знать, и нам это не нужно знать . Нам нужно только сравнить асимптотические состояния до и после взаимодействия. Чтобы сделать это, нам нужно выполнить некоторую аппроксимацию, обычно пертурбативное разложение (что приводит к диаграммам Фейнмана, неправильно называемым «виртуальным частицам» и тому подобному). Сравнение между состояниями входа и выхода, закодированное в С матрица , достаточно, чтобы предсказать скорости распада, сечения и коэффициенты ветвления для любого процесса, который вы можете себе представить. И эти наблюдаемые — единственные, которые мы можем измерить. В заключение, вещи, которые вы не можете точно определить, являются вещами, которые вы не можете проверить экспериментально.

Этот ответ - только набросок, полный ответ потребовал бы от меня написания книги по этой теме. Если вы хотите узнать больше, я рекомендую вам прочитать любую книгу по QFT, например Peskin & Schroeder, Weinberg, Srednicki и т. д.


* В теории взаимодействия классические уравнения движения нелинейны и не могут быть решены с помощью разложения Фурье, которое производит операторы рождения и уничтожения. В формулировке интеграла по путям мы знаем только, как решать интегралы Гаусса (т.е. свободные поля). Чтобы решить интегралы по путям для взаимодействующих полей, нам все еще нужны приближенные методы, такие как пертурбативные разложения или решеточная КТП. По словам Пескина и Шредера:

Неизвестно точно решаемых теорий взаимодействующих полей более чем в двух пространственно-временных измерениях, и даже там решаемые модели включают в себя особые симметрии и значительную техническую сложность.


Для получения более подробной информации об этом я отсылаю вас к этому превосходному ответу ACuriousMind на другой ваш вопрос.

Мне нравится этот ответ больше всего просто потому, что он не сдерживает суровую реальность, что это интерпретация математики, которая определяет эти объекты.
Что вы имеете в виду под «мы не знаем, как точно их квантовать »?
@GennaroTedesco Что в теории взаимодействия классические уравнения движения нелинейны и не могут быть решены с помощью разложения Фурье, которое производит операторы создания и уничтожения. Согласно Пескину и Шредеру: «Нет точно решаемых теорий взаимодействующих полей более чем в двух пространственно-временных измерениях, и даже там решаемые модели включают в себя особые симметрии и значительную техническую сложность».
Ну, вам не нужно писать квантование в терминах операторов уничтожения и создания; вы можете просто написать интеграл по путям для взаимодействующего лагранжиана. Тогда это было бы неперенормируемым или плохо определенным, или еще что-то, я согласен, но концептуально вы также можете квантовать взаимодействующие теории.
@GennaroTedesco В формулировке интеграла по путям мы знаем только, как решать интегралы Гаусса (т.е. свободные поля). Чтобы решить интегралы по путям для взаимодействующих полей, нам все еще нужны приближенные методы, такие как пертурбативные разложения или решеточная КТП.
Я полностью согласен с тем, что вы, конечно, не можете вычислить интеграл по путям.
Босонеандо: Я не разделяю вашего мнения: «Мы не знаем, что такое фотон… мы не знаем, что происходит во время взаимодействия, мы не можем этого знать, да нам и не нужно это знать».
Bosoneando В моем асимптотически менее экспертном подходе я очень разделяю ваше мнение (повторенное здесь выше Джоном Даффилдом), изложенное в моем PDF-файле, упомянутом в моем последнем комментарии-ответе @Inquisitive намного ниже.

Онтологический ответ

Нет короткого ответа .

Фотон — это именно то, что вы получаете, когда изучаете все наши знания о нем в виде математических теорий, большинство из которых имеют в своих названиях большую букву Q. А потом, наверное, еще что-то, чего мы еще не нашли. Ярлыков нет.

Обоснование, ссылка

Это, по сути, отговорка. На вопрос «как ведет себя фотон?», «что мы знаем о взаимодействии фотона с XXX?» и т. д. было бы легко ответить. Но я полагаю, что на вопрос «что такое фотон» (в смысле «что это такое на самом деле , если отбросить всю математику?») не может быть никакого осмысленного ответа в том же отношении, что и на вопрос «что такое ХХХ» ? (где XXX — любая частица или поле Стандартной модели) не имеет смысла.

Вместо того, чтобы много печатать, я предлагаю интервью Фейнмана, Ричард П. Фейнман - ВЕСЕЛО ВООБРАЗИТЬ (полностью) ; часть, относящаяся к этому ответу (« истинный смысл вещей»), идет с 01:03:00 прямо до конца (резюме: даже если у нас есть верные теории о частицах и, таким образом, мы можем объяснить их эффекты, у нас все еще нет способа объяснить их в повседневных / практических терминах без математики, и их никогда не будет, поскольку под ними нет «мирских» законов). Также частично, в отношении легкого, короткого, отвечает часть, начинающаяся с 17:20 (резюме: трудно описать что-либо полностью в том или ином «разрешении», оно идет все глубже; как я уже сказал, лишь частично связано, но вполне познавательно еще).

РЕДАКТИРОВАТЬ: добавлены сводки.

«Вместо того, чтобы много печатать». Тем не менее, резюме было бы в порядке.
Добавлено, @PeterMortensen.

Чтобы действительно ответить на вопрос, что такое фотон, требуется много страниц, и разные эксперты дают разные ответы. Это видно из интересного сборника статей, объясняющих различные современные взгляды:

Природа света: что такое фотон? Новости оптики и фотоники, октябрь 2003 г.

В моем FAQ по теоретической физике есть запись под названием «Что такое фотон?». Вот краткий отрывок; но чтобы ответить на вопрос более подробно, необходимо прочитать саму запись FAQ. С начала:

Согласно квантовой электродинамике, наиболее точно проверенной теории в физике, фотон представляет собой одночастичное возбуждение свободного квантового электромагнитного поля. Более формально, это состояние свободного электромагнитного поля, которое является собственным состоянием оператора числа фотонов с собственным значением 1.

Чистые состояния свободного квантового электромагнитного поля являются элементами фоковского пространства, построенного из однофотонных состояний. Общий n-фотонный вектор состояния представляет собой произвольную линейную комбинацию тензорных произведений n 1-фотонных векторов состояния; а общее чистое состояние свободного квантового электромагнитного поля представляет собой сумму векторов состояния n-фотонов, по одному на каждое n. Если вклад вносит только 0-фотонный член, мы имеем темное состояние, обычно называемое вакуумом; если вклад вносит только однофотонный член, мы имеем один фотон.

Отдельный фотон имеет те же степени свободы, что и классическое вакуумное поле излучения. Его форма характеризуется произвольным ненулевым действительным 4-потенциалом A(x), удовлетворяющим свободным уравнениям Максвелла, которые в калибровке Лоренца принимают вид А ( Икс ) знак равно 0 , выражающий нулевую массу и поперечность фотонов. Таким образом, каждому такому A соответствует чисто фотонное состояние |A>. Здесь A(x) — не оператор поля, а амплитуда фотона; фотоны, амплитуды которых различаются не зависящим от x фазовым множителем, одинаковы.

И почти с конца:

Разговоры о фотонах обычно ведутся непоследовательно; почти все, что говорится в литературе о фотонах, следует воспринимать с недоверием. Есть даже такие люди, как лауреат Нобелевской премии Уиллис Э. Лэмб (открыватель лэмбовского сдвига), которые утверждают, что фотонов не существует. См. ближе к концу
http://web.archive.org/web/20040203032630/www.aro.army.mil/phys/proceed.htm Ссылка, упомянутая там в конце, появилась как WE Lamb, Jr., Anti-Photon , Прикладная физика Б 60 (1995), 77-84. Это, вместе с другой ссылкой, упомянутой Лэмбом, перепечатано в WE Lamb, Jr., Theterpretation of квантовая механика, Rinton Press, Princeton 2001.

Я думаю, что наиболее подходящая интерпретация «наблюдаемого» фотона, используемого на практике (в отличие от фотона, формально определенного, как указано выше), — это когерентное состояние низкой интенсивности, произвольно разделенное на временные срезы, несущие энергию час ν знак равно ю , энергия фотона на частоте nu и угловой частоте omega. Такое состояние состоит в основном из вакуума (который нельзя наблюдать напрямую, поэтому им обычно можно пренебречь), а вклады многофотонных состояний пренебрежимо малы по сравнению с вкладом одиночных фотонов. При таком понятии фотона большинство фактически проведенных экспериментов имеют смысл, хотя оно и не объясняет квантовую случайность процесса обнаружения (который исходит от квантованных электронов в детекторе).

Смотрите также слайды моих лекций здесь и здесь

Для меня столь же загадочны и другие элементарные частицы. Это связано с их неинтуитивным характером.

Наивная онтология мира

Мы, люди, растем в мире объектов . Эти объекты имеют массу и объем, у них есть дискретные границы. Наш мозг имеет обыкновение рассматривать объекты как отдельные вещи , и каждая вещь может, например, быть поднята и иным образом проверена органами чувств. Вещи соответственно обозначаются обычными существительными.

Затем есть жидкости, которые почти как вещи, но не совсем. У них действительно есть масса и объем, и с ними можно взаимодействовать через чувства (хотя осознание того, что воздух не является вакуумом, замечая сопротивление воздуха, является нетривиальным скачком понимания), но они не являются отдельными. Они сливаются друг с другом и могут быть произвольно разделены, в отличие от вещей , так что вы никогда не сможете понять, что такое жидкость. Соответственно, мы знаем их под неисчисляемыми существительными и понимаем, что в конечном счете имеет смысл мыслить частями единого вселенского флюида . Универсальная жидкость становится единственной вещью , так как другие подразделения спорны, за исключением особого случая капель или количеств, изолированных в сосудах, гдевещность навязывается искусственно (т. е. вы можете притворяться , что кофе и молоко — разные вещи, пока они находятся в разных чашках, но в тот момент, когда вы позволяете им соприкоснуться, это притворство разваливается).

Все, кроме этого, мы мыслим как явления . Например, огонь — это то, что случается: это не то, что можно взять и чем можно манипулировать (вы можете только подобрать топливо, а огонь странным образом цепляется за него), и это не то, что вы можете заключить в контейнеры и разделить на части. или комбинировать (хотя топливо может быть как вещью, так и жидкостью, и на нем можно работать, иногда с огнём). То же самое для звука, света, температуры и подобных понятий.

Конечно, теперь мы знаем, что огонь — это всего лишь плазма, т.е. жидкость, которая очень быстро «портится» во что-то другое, и что на самом деле ее можно поймать в ловушку с помощью правильного экзотического сосуда. Таким образом, воображение может быть принуждено принять огонь как флюид , хотя в повседневной жизни он все еще выступает как явление , поэтому он не является истинно интуитивной вещью.

Теория атома

Когда возникла теория атома, я уверен, что греки в какой-то момент ожидали, что «атомы» будут иметь некоторую массу, форму и размер, т.е. быть вещами . Таким образом, используется хитрый концептуальный трюк: для небрежного глаза песок кажется жидкостью , поскольку его количества кажутся свободно сливающимися и разделяющимися (обратите внимание, что песок неисчислим). Но при ближайшем рассмотрении песок оказывается просто кучей крошечных объектов, которые сами по себе явно являются вещами .

Затем пришло осознание того, что мир имел бы смысл, если бы все было своего рода песком , состоящим из множества крошечных частиц, которые сами по себе являются вещами. Это хорошо, потому что объединяет флюиды и вещи: флюиды — это только кажущийся класс, а в глубине души они суть все вещи, что хорошо. Для нас, людей, вещи очень интуитивно понятны, и может быть проще рассуждать о куче миллионов крошечных вещей, чем об одном кусочке жидкости, потому что жидкости очень странные. К счастью, оказалось, что и жидкости, и вещи состоят из частиц, и эти частицы действительно кажутся настоящими вещами .

Здесь начинаются предостережения. Молекулы — это не совсем ньютоновские твердые тела. Они ведут себя почти как они: например, они могут иметь массу и объем. Почти все они могут быть разбиты на части, но если принять во внимание очень жесткое правило разделения молекулы и камня, это уже начинает выглядеть забавно. У них есть граница, и они отталкиваются друг от друга... Но будьте осторожны, чтобы не ударить их слишком сильно, иначе они странным образом сольются вместе (в отличие от камней). Но худшая часть - это граница, которая является всего лишь фальшивой границей: радиус Ван-дер-Ваальса - это не двоичный разделитель "может/не может пройти", а является следствием уравнения непрерывной силы. На самом деле быть немного внутри молекулы не намного труднее , чем немного снаружи . Сравните быть немного внутрискалы -- невозможно.

Кстати, мне кажется интересным, что греки придумали теорию атома, а не теорию жидкости, согласно которой все твердые объекты на самом деле представляют собой жидкости в некотором временном состоянии жесткости. На ум приходят различные теории элементов, но они не делают правильных физических наблюдений: можно было бы наблюдать, как отдельные куски железа можно расплавить и беспорядочно соединить, а затем сделать вывод, что, несомненно, можно расплавить любую вещь , поэтому есть только кажущиеся вещи, и все по сути является жидкостью. Возможно, это потому, что эта теория жидкостей делает мир более запутанным, а не менее.

Субатомные частицы

Молекулы, как оказалось, представляют собой просто небольшие скопления вещей — ведь когда вы разбиваете вещь, в результате должны получаться более мелкие вещи ? Вскоре мы узнали об атомах, а затем и о частях атома. На этом мы остановимся, поскольку, насколько мне известно, ни одна из элементарных частиц не делится на другие составляющие. Фотоны — одна из таких элементарных частиц.

Притворство вещи можно поддерживать для атомов и молекул с помощью таких устройств, как объем Ван-дер-Ваальса. Для элементарных частиц это притворство безнадежно. Как было снова и снова показано, элементарные частицы не только не имеют объема, они явно не имеют объема: если бы они были, то физика явно не работала бы, и вы получали бы такие вещи, как «поверхности» вращающихся электронов. Быстрее скорости света. Затем было сделано наблюдение, что мир имел бы смысл, если бы только эти частицы были точками.

Конечно, никто на самом деле не знает, что такое точечная масса. Никто никогда не видел ничего подобного (ну, кроме того, что мы хотим окрестить точечной массой в первую очередь). Его последствия кажутся странными: например, его плотность бесконечна, и теоретически вся Вселенная может быть сжата в одну точку. Если бы частицы были вещами , подобные безумия были бы благополучно предотвращены: камни нельзя сжимать произвольно, даже с бесконечной силой.

Не сжимаемость, кстати, еще одно свойство вещей . Оказывается, даже в мягких вещах, таких как губки, есть просто карманы для воздуха. После того, как все отверстия выдавлены, вещь не может быть сжата дальше: жидкости вежливо поддерживают этот принцип на словах, хотя вы можете сказать (например, по эксперименту с водой в шприце), что их сердце на самом деле не в этом, и газам просто наплевать -- еще один аспект , в котором жидкости странны и непохожи на вещи . Или, по крайней мере, для наивного наблюдателя, не имеющего доступа к экстремальным энергиям, необходимым для наших современных физических экспериментов.

На субатомном уровне интуиция полностью разрушается. Вы можете создавать аналогии , например, с нитками и горшками с водой, но вы никогда не сможете представить себе, что такое частица с точки зрения того, какие предметы из повседневной жизни. Вселенная сыграла с нами очень злую шутку, в том, что она одна , но она такова, что на макроуровне, на котором мы поневоле начали ее понимать, она совсем другая , без подобия той самой . способ быть увиденным. Тогда мы обречены расти, ожидая и привыкая к другому пути ., только для того, чтобы взять физику 201 в колледже и узнать, что все, что мы знаем, является иллюзией, и никакая интуиция невозможна для истинной природы мира. Интуиция — это понимание, основанное на опыте: кто может познать субатомное? В лучшем случае мы можем испытать экспериментальную аппаратуру .

Нисходящий подход к пониманию Вселенной терпит неудачу, и именно на субатомном уровне.

Версия снизу вверх

Можно спорить об истинном значении интуиции, но я думаю, что некоторое здравомыслие можно восстановить, вместо этого начав все сначала и установив все правильно. Мы можем забыть весь наивный багаж о вещах и жидкостях , стереть все с доски и начать с фундаментальной истины, что в мире есть частицы . Частицы имеют импульс, они точки, они взаимодействуют друг с другом и с вакуумом определенным образом, описанным в квантовой механике. Они элементарны и не состоят из каких-либо более мелких единиц. Таким образом, фотоны являются одной из таких частиц с особыми свойствами, описанными в другом месте (я не буду повторять их, поскольку в своем вопросе вы прямо сказали, что эти описания не то, что вам нужно).

Когда действительно большое количество частиц действует вместе, на макроуровне возникают некоторые причудливые явления, такие как «объемы» и «переходы состояний». Вы не можете интуитивно понять эти причуды, исходя из наших знаний о частицах . Но логически , т.е. если вы следите за математикой, вы знаете, что это простое и прямое следствие, хотя и не интуитивное.

К сожалению, эта интуиция снизу вверх не очень полезна. Вся наша повседневная жизнь связана с макро явлениями. Многие интересные вещи во Вселенной (в основном, все дисциплины, кроме физики элементарных атомов) имеют макромасштаб. Можно ожидать, что после изучения физики мир станет легче понимать, но изучение восходящей интуиции только все усложняет. Я подозреваю, что даже субатомная физика не намного проще, поскольку вся реальная работа выполняется с помощью математики, а не интуиции.

Итак, в заключение, на этот вопрос нельзя ответить удовлетворительно. Есть два способа понять такой вопрос, как «что такое X»:

  • «Расскажите мне о существенных свойствах X»: для фотона нет недостатка в различных текстах, и даже на этом сайте есть вполне полезные ответы, которых вам не хватает.
  • «Помогите мне интуитивно понять X»: как я уже сказал, никакая интуиция невозможна без разрушения всей интуиции, которую вы создали за свою жизнь. Если вы все-таки разрушите его, интуиция вас не удовлетворит и только вызовет у вас головную боль.

Но при этом фотон — это элементарная частица. Он ведет себя как точка. Он имеет импульс и движется со скоростью света (подразумевая, что он не может остановиться). Он имеет связанную электромагнитную волну. Энергия, переносимая этой волной, квантуется. Фотон может взаимодействовать с другими молекулами, например, поглощаясь и испускаясь; с достаточным количеством энергии вы можете создать их «с нуля», и они, кажется, всегда несут с собой пакеты энергии.

Можно задаться вопросом, не являются ли все частицы просто некоторой формой или расположением дискретных битов энергии, которые при агрегировании определенным образом приводят к появлению или видимости точечных масс (или я должен сказать «масс»?) и частиц, и если отдельные планковские единицы энергии действительно являются основой всего остального во Вселенной, и, возможно, фотон очень близок к тому, как эти «энергии» выглядят «само по себе». Возможно, это ближе к тому, о чем вы спрашивали, но на данный момент я твердо запутался, поэтому я остановлюсь здесь.

+1 Я не уверен, что это прямо отвечает на вопрос, но это определенно подразумевает достойную и очень интересную критику неявного требования «интуиции» в вопросе ОП. Я сам не рискнул ответить, потому что выводы, к которым я пришел, твердо помещают меня в категорию тех, кто ничего не может сказать ОП (по его определению). Мне особенно понравилось ваше четкое описание нашей врожденной склонности к поиску «вещественности». Большая часть наших мыслей — это просто эволюция поведения, отражающая нашу эволюционную историю, и не обязательно полезная в мирах, которые мы сейчас исследуем как физики.
Между прочим, и это интересно, хотя иногда мы можем восстановить контакт с представлениями о «вещности», которые, как нам кажется, запрограммированы на любовь довольно абстрактными способами: я говорю о вигнеровском понятии частицы как об «атомарных» подпространствах системы. пространства квантовых состояний, которые остаются инвариантными неприводимым представлением группы Пуанкаре. Это не «вещи», как мы запрограммированы, чтобы их понимать, но за довольно тонкой завесой абстракции они обладают большинством свойств, которые вы описали в первом разделе, которые мы изначально считали «вещью».
Возможно, греки придумали атомы, потому что у них были такие проблемы с бесконечностью. Например, парадоксы Зенона. Атомы — способ удержать жидкость от бесконечной делимости.

Кто может сказать мне, что такое настоящий фотон? Или отошлите меня к какому-нибудь авторитетному информативному определению, которое принято и которому доверяют физики элементарных частиц? Я говорю все это, потому что считаю, что это имеет первостепенное значение. Если у нас нет ясного представления о том, что такое фотон, у нас нет основания...

Как мы можем понять образование пар, если мы не понимаем, что такое фотон? Или электрон? Или электромагнитное поле? Или все остальное? Все начинается с фотона.

Я думаю, что это действительно философская проблема, а не проблема физики. Никакой ответ не удовлетворит вас, потому что вы задаете вопрос, на который невозможно дать окончательный ответ: в чем сущность вещи?

Точно такая же проблема существует с каждым понятием человеческого мышления не только в науке (Что такое энергия? Что такое время? Что такое цвет? Что такое сознание?...), но и в гуманитарных науках (Что такое любовь? Что такое красота? ? Что такое счастье?...). В каждом случае, чем больше мы пытаемся определить что-то, чем более неуловимым оно становится, тем меньше мы, кажется, по-настоящему понимаем, в чем состоит его сущность. И когда мы думаем, что уловили его, появляется какое-то новое свойство, которое снова приводит наше понимание в беспорядок.

Я согласен с AnoE (возможно, потому, что я ученик Ричарда Фейнмана), что вещи могут быть поняты только как сумма их свойств, их взаимоотношений с другими вещами .

В жизни не обязательно знать, что такое любовь, чтобы испытать ее, или знать, что такое справедливость, чтобы поступать справедливо или признавать несправедливость. Единственное определение, которое мы можем дать, состоит в том, чтобы обобщить наш опыт с вещью в одну или несколько идеализированных «моделей», которые изолируют черты, которые мы считаем «существенными».

Точно так же нет необходимости иметь окончательное определение фотона как твердого основания, прежде чем мы сможем изучать свет или разрабатывать мощные теории, такие как КЭД. Адекватно рабочее определение или модель, которая позволяет нам идентифицировать и согласовывать общий опыт и свойства, которые мы исследуем.

История науки показывает, что понятия, которыми мы пользуемся, постепенно совершенствуются на протяжении десятилетий или столетий, в частности вопрос «Что такое свет?» Это отсутствие окончательного определения не помешало нам разработать сложные теории, такие как КЭД и общая теория относительности, которые позволяют нам предсказывать с поразительной точностью и расширять наше понимание того, как устроена Вселенная.

«Фотон», «электрон» и «магнитное поле» — это только наши модели вещей, которые мы находим во Вселенной, которые помогают нам предсказывать и находить отношения между вещами. По словам Элиаса, эти модели по необходимости являются приблизительными понятиями . Они не то, что существует на самом деле. Неизбежно, что они будут меняться по мере того, как мы уточняем наши приближения, пытаясь вписать новые свойства, новые наблюдения в рамки нашего понимания, наших теорий.

приблизительные понятия ? которого ? какой-то реальности ? Я не соглашусь, если вы не скажете, что они являются приближением к следующей ступени человеческого знания. Это концепции в теории, которые отвечают на ряд вопросов. Это все, и это много.
Я имею в виду, что рабочая модель не обязательно должна быть реалистичной во всех своих атрибутах. Я согласен с тем, что модель индивидуальна для теории. Но я думаю, что мы можем классифицировать модели только как полезные , а не как истинные . Максвелл разработал свои уравнения на основе модели молекулярных вихрей, которая сегодня выглядит нелепо, но эта модель работала для него. Я думаю, что это согласуется с вашим окончательным определением: фотон — это понятие в теории. Чтобы конкретизировать концепцию, нам нужно конкретизировать теорию. Как и в случае с Максвеллом, когда теория завершена, мы можем отказаться от модели.
Можно ли это описать как бесконечный процесс уплотнения понятия в пространстве человеческого языка, который по своей природе представляет собой набор эволюционирующих приближений?
@danielAzuelos: хорошо, но сходимость от приближений к идеалу включает концепцию реальности, которая не очень полезна и с ней немного сложно справиться. Осторожность - добродетель в этом деле...

Меня раздражают определения фотона, описанные в вопросе. Дело не в том, что они неправы, а в том, что они ввели меня в заблуждение, как если бы они мешали мне понять, что такое фотон. Ниже то, что я думаю сейчас. Это, конечно, не новая физика, и каждая интерпретация субъективна. Я пройду через это, введя несколько антитез.

1. Фотоны не дискретны

Такие термины, как «частица», «квант света» или «единица обмена энергией», наводят на мысль, что фотоны — это нечто дискретное и внезапное. Второе квантование дополняет эту идею. Например, при вторичном квантовании гамильтониан одного состояния (скажем, конкретной стоячей волны в полости) можно записать как

ЧАС знак равно ю ( а а + 1 / 2 )

Это также гамильтониан для гармонического осциллятора. Следовательно, мы можем легко записать «волновую функцию» этого состояния как Ψ ( д ) и гамильтониан с классической кинетической энергией типа п 2 и потенциальная энергия, как д 2 условия. Мы можем записать эту волновую функцию в виде линейной комбинации,

Ψ ( д , т ) знак равно н с н ( т ) ψ н ( д ) ,

и мы понимаем, что динамика фотонов не так уж отличается от динамики электронов. В середине квантовой динамики (то есть между измерениями) может быть любой вид волнового пакета, описываемый формулой Ψ ( д , т ) или коэффициенты линейной комбинации с н ( т ) . Следовательно, число фотонов не дискретно, и они не обмениваются мгновенно дискретными количествами.

Вместо этого все, что есть, это поле, и оно подвержено типичной квантово-волновой эволюции. Это поле связано с материей.

2. Квантование не уникально

Давайте обсудим две поперечные моды распространяющегося фотона (на самом деле есть еще две, продольная и энергоподобная, но это выходит за рамки). Часто говорят, что фотон имеет угловой момент ± , что соответствует световым частицам с круговой поляризацией. Это приводит к спинорному представлению фотона.

[ Ψ л ( д ) Ψ р ( д ) ]

Однако в некоторых приложениях лучше анализировать только линейно поляризованные фотоны ( Ψ Икс , у ( д ) знак равно 1 2 ( Ψ л ( д ) ± я Ψ р ( д ) ) ). Теперь легко увидеть, что, как и в случае с вращением электрона, была выбрана только предпочтительная система отсчета, и в выборе этих дискретных координат нет ничего особенного. (Конечно, в выборе координат есть что-то особенное: физическая интуиция хорошо описывает задачу.) Но на самом деле я думаю, что даже поперечность поляризации — это выбор точки отсчета.

3. Коллапс волновой функции создает кажущуюся дискретность

Скажем, в нашем глазном рецепторе возбуждается молекула красителя, которая впоследствии меняет свою форму, и передается нервный импульс. Такой процесс напоминает квантовое измерение, поскольку в нем задействовано очень много неконтролируемых степеней свободы при высоких температурах, и происходит явление, называемое декогеренцией. Таким образом, если ранее волновая функция фотона была ( 1 2 | 1 > + е я θ 1 2 | 0 > , эффективная волновая функция (интегрирующая макроскопические степени свободы) находится в дискретном состоянии с вероятностями, заданными их амплитудами. Вот почему фотоны можно увидеть и услышать как щелчки. С долей скептицизма звук возникает из-за коллапса волновой функции :)

4. Фотоны дальнего и ближнего поля различаются

Часто говорят, что фотон имеет определенную энергию и импульс, которые должны сохраняться (т. е. он следует дисперсионному соотношению Е знак равно к и один фотон, который попадает в детектор, всегда имеет это (E,k). Но, например, существуют фотонные кристаллы, в которых энергии фотонов имеют запрещенную зону, а фотоны обладают массой (нелинейные дисперсионные соотношения). Опять же, можно квантовать уравнения Максвелла в фотонном кристалле, выбрав несколько состояний, и отнести частицы к этим состояниям. Здесь также можно говорить о фотонах и даже сказать, что они имеют массу, поскольку их уравнения движения ведут себя так, как будто они имеют массу.

Однако, поскольку измерение обычно проводится в дальнем поле, где фотоны асимптотически свободны, фотоны измеряются как Е знак равно к .

5. Режимы не уникальны

Теперь представьте больше режимов, чем раньше. Теперь волновая функция Ψ ( д 1 , д 2 д Н ) . Теперь представьте, что вы создаете линейную комбинацию этих режимов. д я знак равно Дж А я Дж д Дж максимально локализовать их. На самом деле, давайте локализовать в той мере, в какой один режим д будет соответствовать определенному месту в пространстве. Теперь у вас есть «волновое соединение» фотона, которое дает амплитуду вероятности фотонного поля в различных положениях пространства.

Ψ ( р 1 , р 2 , р Н )

Ограничившись N координатами, описывающими фотон примерно вокруг позиций ( р 1 р Н ), мы фактически наложили ограничение энергии на наши уравнения, и все в порядке.

Теперь представьте себе расширение этого процесса до континуального предела (далеко не тривиального) и включение взаимодействия света с материей, и мы столкнулись с проблемой перенормировки и всеми действительно сложными и хардкорными вещами.

Учитывая все это, хочется из практических соображений и ради физической интуиции вернуться к вторичному квантованию и говорить об одном фотоне в моде 15. Другими словами, вторичное квантование и разговоры о частицах как возбуждениях гармонических осцилляторов — это просто инструменты, созданные физической интуицией и для нее. Но если кто-то хочет понять, что такое фотон, ему нужно заглянуть под капот.

Вы вносите путаницу среди стольких разных вещей, смешивая прежде всего QM и QFT (и что с тем фактом, что квантование не уникально?). Что же такого неудовлетворительного в классической КТП-описании квантования электромагнитного поля?
@GennaroTedesco Я смешал здесь QM и QFT, потому что хотел показать, что самые основы квантовых полей можно довольно хорошо понять с помощью обычного QM. (Но, по сути, единственная ссылка на КТП, которую я делаю, касается перенормировки, остальное — некоторая форма резонаторной КЭД.) Но я могу согласиться, что это, возможно, сбивает с толку :) И, возможно, есть также некоторые путаницы, я был бы рад, если бы вы могли бы уточнить. Однако это то, что касается моего текущего понимания фотонов, и пока что я нахожу это наиболее удовлетворительным. Я изменю мнение, как только соберу больше знаний.
Дело в том, что в КМ вообще нет фотона; она появляется только как силовой носитель электромагнитного поля в КТП (какую бы процедуру квантования вы ни выбрали), которую не следует путать с дискретизацией энергии в таких процессах, как поглощение, излучение и все остальные. Тогда вопрос о том, правильно ли поставлено описание КТП, является вопросом другого рода, но КМ не вызывает никаких противоречий или путаницы только потому, что в ней вообще не участвует фотон.
@GennaroTedesco Я занимался КМ-обработкой степеней свободы электромагнитного поля. Квантовая эволюция, которую я описываю, может быть примером как таковая в уравнении 21 этой статьи nano-bio.ehu.es/files/articles/… Теперь связи даны ad hoc в этой формулировке, и я согласен с тем, что для их правильного получения необходимо выполнить правильную КТП с перенормировками (за пределами меня). Итак, что я сделал, так это использовал КЭД-систему модельного резонатора, чтобы упростить описание фотона. Я думаю, что ваша линия QM против QFT без фотонов в QM слишком строгая.

Я собираюсь начать свой ответ со ссылки на другой: что более фундаментально, поля или частицы?

Фотон - это всего лишь частный случай того, что изложено в этом ответе. Цитирую Дэниел Санк:

Рассмотрим струну скрипки, имеющую набор мод колебаний. Если вы хотите указать состояние струны, вы перечисляете режимы и указываете амплитуду каждого из них, например, с помощью ряда Фурье.

смещение струны ( Икс ) знак равно Режим  н знак равно 0 с н [форма режима  н ] ( Икс ) .

Колебательные моды подобны квантовым собственным состояниям, а амплитуды с н подобны числу частиц в каждом состоянии. С этой аналогией первая нотация квантования, где вы индексируете частицы и указываете состояние каждой из них, похожа на индексацию единиц амплитуды и спецификацию моды каждой из них. Это явно задом наперед. В частности, теперь вы понимаете, почему частицы неразличимы. Если частица — это просто единица возбуждения квантового состояния, то, как и единицы амплитуды вибрирующей струны, бессмысленно говорить, что частица имеет идентичность. Все единицы возбуждения одинаковы, потому что это всего лишь математические конструкции для отслеживания того, насколько возбуждена та или иная мода.

Лучший способ указать квантовое состояние — перечислить каждое возможное состояние и сказать, насколько оно возбуждено.

Фотон именно таков: единица возбуждения (1) моды электромагнитного поля.


Основная проблема с фотоном заключается в том, что люди пытаются слишком упростить его. Это имеет корни в истории. На заре квантовой механики частицы и, в частности, фотоны использовались для объяснения «частичных свойств света». В современном представлении о квантовой теории поля эта картина заменяется тем, что описывает Даниэль Санк в связанном вопросе.

Как таковой фотон сложен . Это не волновой пакет и не маленькая точечная частица. Теория поля объединяет обе эти картины. Тогда реальные фотонные волновые поля являются суперпозицией этих фундаментальных возбуждений и могут отражать как поле, так и поведение частиц. Ответ на следующий вопрос OP...

Как мы можем понять образование пар, если мы не понимаем, что такое фотон? Или электрон? Или электромагнитное поле? Или все остальное?

... лежит там. Если вы хотите знать, что происходит с реальными физическими объектами, вы уходите от фотонов. Однофотонные состояния в природе редки, если вообще не существуют.


Так что же такое фотон ?

Цитата из вопроса:

[...] «фотон есть возбуждение фотонного поля». Это ничего мне не говорит.

Это говорит о многом, математический формализм очень ясен, и многие люди пытались объяснить это в ответах здесь и в других местах.

[...] потому что создается впечатление, что фотоны постоянно возникают и летают туда-сюда, прилагая силу. Эта концепция присутствует и в статье о фотонах в Википедии. Это неправда. Как сказала Анна, виртуальные частицы существуют только в математике модели. Итак, кто может сказать мне, что такое настоящий фотон? [...]

Проблема здесь действительно заключается в отношении между математическим формализмом и «реальностью». «Настоящий» фотон — это не вещь, фотон — это математическая конструкция (которая была описана выше), и мы используем ее (успешно) для описания экспериментальных результатов.


(1) любезно предоставлено DanielSank.

Когда Макс Планк пытался понять проблему излучения черного тела, он добился успеха только тогда, когда предположил, что электромагнитная энергия может излучаться только в квантованной форме. Другими словами, он предположил, что существует минимальная единица света, которая может быть испущена. Предполагая это, он, конечно, нашел Е знак равно час в куда час есть постоянная Планка.

В 1905 году Эйнштейн отнесся к этому серьезно и предположил, что свет существует в виде этих фундаментальных единиц (фотонов) с их энергией, определяемой выражением час в куда в это частота излучения. Этот фотон объяснил многие экспериментальные результаты и придал свету корпускулярно-волновой дуализм.

У многих вещей есть минимальный «кусок»: планковская длина, ниже которой расстояние становится бессмысленным, кварк, глюон и другие фундаментальные единицы материи, планковское время (предположительно минимальное измерение времени, это время, необходимое для света). пройти в вакууме расстояние, равное одной планковской длине).

Итак, что такое фотон?

Это минимальная «единица» света, фундаментальная часть. Я бы сказал, атом света, но это не совсем правильное изображение. («Кварк» света?)

Также важно помнить, что мы не можем «видеть» фотон так же (ну, даже в большей степени), как мы не можем «видеть» кварк. То, что мы знаем о них, получено из экспериментов и расчетов, и поэтому на самом деле не существует их физической картины, как это верно для большей части квантовой механики.

Фотон — это всего лишь единица возбуждения электромагнитной моды.
Мы можем приблизиться к тому, чтобы увидеть фотон, чем вы думаете: nature.com/news/…
Большинство физиков, в том числе теоретики струн, категорически не одобряют, что длина ниже планковской длины не имеет смысла.
Как вы понимаете, что фотон является «минимальной» единицей, если он может иметь любую энергию в непрерывном диапазоне энергий?
@ user50229, фотон — это наименьшая возможная единица энергии на заданной длине волны.
@heather Если вы зафиксируете длину волны, то возможна только одна «единица» энергии, для фотона с фиксированной длиной волны нет диапазона энергий. Так что по-прежнему не имеет смысла говорить, что фотон — это наименьшая возможная единица энергии на данной длине волны. Это единственная единица.
@heather Я думаю, вы могли бы сказать, что фотон несет наименьшее количество энергии, возможное на данной длине волны, поскольку большее количество энергии потребует более одной единицы (больше фотонов).
@ user50229, прошу прощения, если я неясно выразился, я имел в виду то, что вы сказали (что фотон несет наименьшее количество энергии, возможное на данной длине волны).
Длина волны — это просто удобный термин. Волны нет. Расстояние, которое проходит один фотон за одно колебание, называется длиной волны. Фотон с длиной волны 500 нм на самом деле имеет частоту более 600 000 000 000 000 колебаний в секунду. Когда он движется со скоростью света, одно из этих 600 триллионов колебаний совершается за 500 нм.
@heather: Согласны ли вы с тем, что мы можем говорить об этой единице энергии только в контексте взаимодействий, то есть когда фотон поглощается или испускается? Пока свет распространяется, мы не знаем, существуют ли фотоны.

В любой концепции физики существует дихотомия между моделью и физической системой. На практике мы забываем об этой дихотомии и поступаем так, как будто модель и физическая система — одно и то же. Многие ответы на вопрос "что такое фотон?" будет отражать это отождествление модели и физической системы, т. е. фотон есть идеальная точечная частица; фотон квант поля; фотон _линия на диаграмме Фейнамна и т. д. Эти определения фотона глубоко укоренены в моделях. Наша склонность отождествлять модель и физическую систему коренится в ложном предположении, что интуиция и образы, которые мы развиваем для понимания модели, в равной степени применимы и к физической системе. Мы убеждаем себя, что маленькое белое пятнышко, мчащееся в трехмерном пространстве со скоростью света в нашей голове , — это фотон, тогда как на самом деле это образ, связанный с моделью.

В свете этого есть два основных способа ответить на вопрос «что такое фотон?» Первый способ — обратиться к модели и сказать, что «фотон — это концепт X в модели Y». Многие пользователи выбрали этот путь. Второй способ — сослаться на эксперимент и сказать, что «фотон — это то, что отвечает за это значение данных». Я предпочитаю этот путь, когда отвечаю на вопрос "что такое ___?" потому что это позволяет избежать предположения, что модель и физическая система идентичны. Применительно к фотону я бы сказал: «Фотон — это пакет электромагнитного излучения, который удовлетворяет Е знак равно час ν и это наименьший пакет электромагнитного излучения».

Если вы недовольны обоими типами определений, вам не повезло. Наши модели навсегда останутся в наших головах, а физический мир, который они так точно описывают, всегда будет недосягаем.

Это хороший момент, который, вероятно, будет полезен любому, кто сбит с толку этим разнообразием описаний.

Моя запись:

Для свободного или слабо взаимодействующего электромагнитного поля, имеющего излучение в некоторой области с определенной частотой и энергией, существует минимальное ненулевое количество энергии, которое можно добавить к полю или отнять от него. Это количество и есть «фотон».

Теперь мелкий шрифт:

  • Конечно, многие другие ответы здесь более точны, и я думаю, что многие из них также проницательны, но я воспринял вопрос как «объясните это, как ребенок, но правдиво», и попытался подойти как можно ближе. возможно к этому.

  • Как уже отмечали другие, фотон не всегда используется последовательно, но практически для каждого использования, я могу думать, что приведенное выше утверждение верно (если вы думаете, что у вас есть контрпример, пожалуйста, укажите мне на него). Я говорю «практически», потому что единственное исключение, о котором я могу думать, это так называемый «виртуальный фотон». Тем не менее, я думаю, что эта терминология в любом случае чрезмерно используется неспециалистами, и ее следует избегать или, по крайней мере, следует обсуждать отдельно.

  • Между «сильной» и «слабой» связью есть стандартное точное определение среди физиков, но на самом деле переход между свободными фотонами и возбуждениями материи к сильно связанным возбуждениям, таким как поляритоны, происходит плавно, и ни в одной точке нет резкого качественного изменения.

  • Экспериментально требование «определенной частоты» часто слегка ослабляется до «четко определенной частоты», потому что любой реальный источник света всегда имеет некоторую конечную спектральную ширину. Это один из вопросов, который иногда вызывает небольшое различие между экспериментальным и теоретическим представлениями о «фотоне».

  • Это определение, сформулированное в терминах энергии определенной части поля, поначалу трудно согласовать с картиной «частиц бильярдного шара», которую вы могли бы себе представить, когда фотоны представляют собой дискретные объекты, которые летают вокруг и отскакивают от вещей. . Это просто потому, что эта картина во многих случаях сильно ошибочна. В некоторых очень специфических ситуациях (например, комптоновское рассеяние) вам может сойти с рук это. Однако это так часто вводит в заблуждение, что, вероятно, было бы лучше полностью отбросить его, пока вы не поймете условия, при которых оно примерно справедливо, что является тонким вопросом, достойным совершенно отдельного обсуждения. В большинстве случаев фотоны совсем не похожи на маленькие «бильярдные световые шары».

Я полностью согласен с ответом флиппифануса . Прежде всего, фотон — полезное понятие , введенное для описания явлений, к которым у нас нет интуитивного подхода, и кроме того, мы не знаем, что такое фотон. Хотя это правда, это не особенно удовлетворительно. Что я хочу добавить к его ответу, так это то, почему было введено понятие фотона.

Долгое время шел спор о том, является ли свет частицей или волной. Ньютон поддержал и развил большую часть «корпускулярной теории света» . Его самым сильным аргументом было то, что свет распространяется по прямым линиям, а волны имеют тенденцию рассеиваться в пространстве. Гюйгенс, с другой стороны, утверждал, что свет представляет собой волну. Волновая теория света могла объяснить такие явления, как дифракция, которые не смогла объяснить корпускулярная теория. Когда Юнг провел свой знаменитый эксперимент с двумя щелями , который показал интерференционные картины, точно такие же, как те, которые известны для звуковых волн или волн воды, вопрос, казалось, был решен раз и навсегда.

[Интересное примечание: волнам для распространения нужна среда, но свет также распространяется через вакуум. Это привело к постулированию так называемого эфира , среды, которая должна была пронизывать все пространство. Однако свойства этого эфира были противоречивы, и никаких экспериментальных подтверждений этому найдено не было. Это сыграло роль в развитии теории относительности, но это уже другая история]

Затем, примерно в 1900 году, Макс Планк смог правильно описать спектр черного тела с помощью того, что он сначала принял за математический трюк: для своих расчетов он предположил, что энергия излучается крошечными порциями, а не непрерывно, как вы предположил бы, если бы свет был волной. Спектр черного тела был одним из самых важных нерешенных вопросов в то время, и его объяснение было научным прорывом. В результате его метод получил большое внимание.

Вскоре после этого Эйнштейн использовал метод Планка для объяснения другой нерешенной проблемы физики: фотоэлектрического эффекта . Опять же, это явление можно было бы описать, если представить свет в виде небольших пакетов энергии. Но в отличие от Планка Эйнштейн считал эти пакеты энергии физической реальностью, которые позже были названы фотонами .

Этот неологизм был во что бы то ни стало оправданным, поскольку к тому времени уже было ясно, что фотоны должны быть чем-то другим, а не просто маленькими бильярдными шарами, как представлял себе Ньютон. Иногда он проявляет волновые свойства, которые нельзя объяснить с помощью классических частиц, иногда он проявляет корпускулярные свойства, которые нельзя объяснить с помощью классических волн.

Вот что мы знаем:

  • Свет излучается дискретным количеством пакетов. Это означает, что существует исчисляемое количество объектов света (которые мы сегодня называем фотонами). Это утверждение, вероятно, является наиболее фундаментальным для идеи фотона. Это также был бы мой «ответ одним предложением», если бы кто-то спросил меня, что такое фотон.

  • Они несут физические свойства, такие как энергия и импульс , и могут передавать их между физическими объектами (например, когда тепловое излучение поглощается и нагревает тело).

  • Поскольку у фотонов есть импульс, они также должны иметь массу .

  • Позже было также показано, что фотоны обладают и другими свойствами, такими как спин

  • Они распространяются по прямым линиям без необходимости в среде (было доказано, что эфира, о котором я упоминал ранее, не существует).

Все эти свойства обычно связаны с частицами. По крайней мере, они показывают, что фотон — это что- то (в том смысле, что совокупность физических свойств обычно квалифицируется как вещь ). Но у фотона есть и свойства, отличные от классических объектов:

  • Когда фотоны распространяются, они демонстрируют дифракцию , преломление и интерференцию.

  • Энергия и импульс фотона соответствуют длине волны и частоте света, которые определяют поведение интерференции и дифракции.

Суть в том, что фотон — это не волна и не частица, а квантовый объект , обладающий как волновыми, так и корпускулярными свойствами. В целом можно сказать, что свойства, подобные частицам, преобладают, когда вы смотрите на

  • небольшое количество фотонов

  • их взаимодействие с материей (например, упомянутый вами процесс образования пар)

  • высокие энергии

в то время как волнообразные свойства преобладают, когда вы смотрите на

  • большое количество фотонов

  • их распространение в пространстве

  • низкие энергии

Я думаю, что это все, что можно сделать с классическими аналогиями. Фотон — это то, о чем нам говорят его свойства и поведение, а все остальное — неполная аналогия. Лично мне нравится представлять фотоны (как и в случае с любой другой визуализацией, это ни в коем случае не правильно, но во многих ситуациях это хорошо работает и помогает взять себя в руки) в виде маленьких, твердых, дискретных частиц, которые движутся в пространстве подобно волнам.

Я не согласен с тем, что «свет излучается дискретным количеством пакетов». Насколько я понимаю, это измерение, которое «схлопывает» волновую функцию фотона, так что фотоны появляются как дискретные щелчки на счетчике фотонов и т. д. Другими словами, во время квантового распространения возбужденного состояния атома многие фотонные состояния приобретают амплитуду . Однако они переплетаются с результатами измерительного устройства таким образом, что дискретность появляется из-за декогеренции. Я бы сказал: «свет измеряется дискретными пакетами».
«Поскольку у фотонов есть импульс, у них должна быть и масса». Бред какой то. Релятивистское понимание импульса очень важно для внутренне релятивистской сущности, не так ли?
Энергия света не является квантованной величиной. Энергия света данной конкретной частоты является квантованной величиной, основанной на формуле w=hv. общая энергия частоты = v света должна быть в N раз больше hv. Но это самоочевидно и, кажется, не имеет большого значения. Мы берем любое количество вещей в качестве инициализации, общая стоимость должна быть в N раз больше этой единицы.
@dmckee Релятивистская масса.

Излучение испускается, когда электрон замедляется, и поглощается, когда он ускоряется в соответствии с известной формулой Лармора. Это излучение представляет собой непрерывное электромагнитное поле. Внутри атома электроны меняют орбиту и быстро ускоряются, в результате чего излучение испускается и поглощается так быстро, что оно проявляется в виде линий спектра. Но помимо спектральных линий, атомы и молекулы излучают на очень многих других частотах из-за различного колебательного движения в них и соответствующего ускорения/замедления, сопровождающего это. Эти фоновые частоты, естественно, будут иметь более низкую частоту, чем частота линейчатых спектров для такая же система. Черенковское излучение, пожалуй, ближе всего к непрерывному спектру.

Поэтому излучение всего вещества состоит из резких спектральных линий на фоне непрерывного излучения. Фотон – это единица энергии излучения, которой обмениваются связанные (не свободные) электроны. Это как валюта денег... и как валюта, фотоны не все одного и того же номинала/энергии. Формула E=nhf дает n как количество обмененных фотонов с частотой f, что приводит к энергии E. Но поскольку f переменная и даже не дискретная, E не дискретно, а фотон.

Фотон также описывается как пакет энергии. Это верно, но означает только минимальную (n=1) энергию определенного цвета, которой можно обмениваться при определенном взаимодействии между атомами. Обычно энергия волн прямо пропорциональна квадрату их амплитуды и не имеет ничего общего с частотой. Чтобы согласовать это с определением фотона, очень необходимого для ядерного взаимодействия, вводится число фотонов n, чтобы компенсировать исходную формулу E=hf. Обратите внимание, что хотя фотон представляет собой пакет энергии, количество энергии в пакете может варьироваться. Например, один синий фотон обладает гораздо большей энергией, чем красный фотон. Гамма-лучи имеют самое высокое содержание энергии из-за их более высокой частоты.

Если вы прочитаете книгу по фотонике, то обнаружите, что слово «фотон» встречается почти в каждой ее строке. Это показывает, насколько важна концепция фотона, несмотря на ее необычное и немного запутанное определение и использование.

Насколько я понимаю, мы согласны:

Мы используем слово Фотон, когда каким-то образом наблюдаем, что материальный объект меняет содержание энергии, импульс и вращение, и в непосредственной близости нет другого материального объекта, с которым можно было бы взаимодействовать.

Мы знаем, что есть магнитная сила и электрическая сила, и что эти силы могут быть объединены с электромагнитной силой, и пространственное и своевременное развитие этих сил позволяет электромагнитным волнам быть очень похожими на другие волны, которые можно наблюдать непосредственно, как волны воды.

Из воды и воздуха мы знаем, что определенные возбуждения могут существовать как плоские или циркулярные волны. Особые формы волны представляют собой вихри или более общие солитоны.

Мы знаем, что солитоны могут существовать как квантованные возбуждения, как в сверхтекучих жидкостях. Так что есть определенная вероятность, что такие солитоны могут быть решениями уравнений Максвелла. Это позволило бы представить себе, что такое Фотон на самом деле.

Пока писал, пришла идея погуглить "фотонный солитон". Я нашел эту открытую статью в качестве отправной точки для меня: Фотоны как солитоны и там: прямой доступ

PS, пожалуйста, не понижайте оценку этого ответа, так как он находится в стадии разработки, и я отреагирую на любой комментарий, чтобы быть более четким. И я не могу комментировать, чтобы другие ответили только сейчас. Спасибо

Этот ответ не учитывает гравитацию… Например. Пусть тело имеет полость. В этой полости тело излучает фотоны и поглощает фотоны. Между излучением и поглощением есть временной интервал. Масса тела состоит из массы материала тела и энергии фотонов в этой полости для внешнего наблюдателя. Существуют ли фотоны в вакууме? Являются ли их пары источником гравитации? Влияют ли фотоны на пространство-время?
@Imyaf: вопрос довольно прост, и ответы на него очень сложны. Я вижу материальные вещи как расположенные конденсаты энергии. Поскольку мы знаем, что материя может разрушаться в ядерных реакциях с испусканием фотонов, захваченные фотоны должны вносить вклад в массу ловушки. Но это можно измерить только для фотонов высокой энергии (гамма).

Друг спросил меня об этом в колледже, и я примерно так ему и ответил.

Экспериментаторы выясняли поведение электрических и магнитных явлений в конце 18-го и начале 19-го веков, и примерно к середине 19-го века все это складывалось воедино. Джеймс Клерк Максвелл внес «последние штрихи» в уравнения, описывающие (классические) электрические и магнитные явления.

Одно из этих уравнений (закон Фарадея) описывает, как изменяющееся магнитное поле может индуцировать электрический ток , а другое уравнение (закон Ампера) описывает, как электрический ток может индуцировать магнитное поле .

Итак, подумайте об электронах, у них есть электрический заряд, если мы «встряхнем» один из них , мы можем создать изменяющееся электрическое поле, которое индуцирует изменяющееся магнитное поле, которое индуцирует изменяющееся электрическое поле и так далее... Эти маленькие пульсации электрических и магнитных полей, индуцирующих друг друга, вот что такое фотоны .

В какой-то момент я прочитал увлекательный рассказ Максвелла о том, как, разработав уравнения для описания электромагнетизма, он обнаружил, что их можно использовать для вывода волнового уравнения. У волновых уравнений есть константа, которая описывает скорость распространения волн, и его полученное волновое уравнение имело вид 1 мю ϵ для этой константы (с мю является проницаемостью и ϵ диэлектрическая проницаемость).

Если я правильно помню, эти значения можно было измерить с помощью экспериментов с электричеством. Что-то вроде подачи известного количества тока через два параллельных провода, они будут генерировать магнитную силу, которая раздвигает провода (если ток в одном направлении, и стягивает их вместе, если он в противоположных направлениях, я думаю?). Измерение результирующей силы может сказать вам значение (я полагаю, только одного из них).

Таким образом, эти значения были измерены, и подставив их Максвеллу, мы получили что-то близкое к скорости света, которую экспериментаторы измеряли с возрастающей точностью примерно в то время (особенно в 1849 и 1862 годах). И это был первый раз, когда кто-то (Максвелл) смог понять, что свет — это какое-то электромагнитное явление. [Глядя вверх, я вижу, что на самом деле Вильгельм Эдуард Вебер и Рудольф Кольрауш в 1855 году заметили единицы мю а также ϵ могли производить скорость, и они измерили их экспериментально и получили число, очень близкое к скорости света, но не сделали того последнего логического скачка, который сделал Максвелл в 1861 году.] (Из статьи в Википедии History of Maxwell's уравнения )

Я не эксперт, но у меня сложилось впечатление, что Максвелл также заметил, что его уравнения кажутся неполными, потому что они предполагали, что скорость света остается постоянной независимо от скорости наблюдателя или излучателя. Обычно люди думают, что работа Эйнштейна по специальной теории относительности разрешала знаменитый нулевой результат экспериментов Майкельсона и Морли с интерферометром в поисках светоносного эфира, но на самом деле Эйнштейн обращался к этой инвариантности, указанной уравнениями Максвелла, если я правильно понимаю.

(Обратите внимание, что прошло много времени с тех пор, как я рассказывал большую часть этого, и у меня есть только степень бакалавра наук в области математической физики, и я не использовал эти знания в течение длительного времени, так что, возможно, я Я ошибаюсь в некоторых деталях, но я думаю, что общий смысл довольно близок.)

Прежде чем даже пытаться спросить, что такое фотон «точно», мы должны спросить: существуют ли фотоны?

Вы можете пройти долгий путь, полагая, что это не так. Атомы, молекулы и кристаллы имеют дискретные состояния, которые определяют квантовую природу материи, поэтому испускают и поглощают кванты энергии, в то время как сама сущность может быть непрерывной, подобно тому, как вино является непрерывной сущностью, которая квантуется только до 70 cl из-за бутылок, в которые она вливается. продается. Квантовая механика использует классическое электромагнитное поле. Волнистые линии на диаграммах Фейнмана, часто называемые фотонами, являются просто графическим обозначением членов в разложении теории возмущений.

Однако остается проблема: как получается, что фотоны поглощаются в очень локальных реакциях? Каким образом протяженная классическая электромагнитная волна может быть поглощена одним атомом? Для меня разумная интерпретация этих явлений состоит в том, что электромагнитное поле описывает вероятность того, что имеет место поглощение/излучение.

По этой причине я теперь убежден, что дискретные фотоны существуют и что волновое уравнение, лежащее в основе уравнений Максвелла, является релятивистским квантовым волновым уравнением, описывающим безмассовые квантовые частицы точно так же, как уравнения Шредингера, Дирака и Клейна-Гордона описывают массивные квантовые частицы. Уравнение электромагнитной волны в моей интерпретации представляет собой безмассовое уравнение Клейна-Гордона, описывающее квантовые частицы, известные как фотоны.

Это не дает ответа на вопрос, что такое фотон. Он предлагает ответ на предыдущий вопрос, существуют ли фотоны.

Это большой «дзэнский» вопрос физики на протяжении веков, спасибо, что задали его. Другие ответы хороши/приемлемы, этот (репутационный рискованный, но искренний/подробный) использует в некотором роде радикально другой угол/подход. Другие ответы смотрят в прошлое, этот будет пытаться сделать почти невозможное предвидение будущего дальновидным, но =--- все еще научно обоснованным способом. Другими словами, сомнительные части можно считать гипотетическими, то есть рассматриваемыми гипотезами, но все они тщательно подкреплены текущими/надежными (в некоторых случаях очень недавними) результатами исследований.

В фотонной истории есть драматический аспект «слепых и слонов», который охватывает многие века. Волновая и корпускулярная природа света обсуждалась даже во времена Ньютона в 17 веке, сейчас около 4 столетий назад, и световые единицы или частицы были названы «корпускулами» в отличие от волновой теории Гюйгенса. Теория Ньютона господствовала над последней около века «отчасти из-за большого престижа Ньютона», хотя Гюйгенс был сформулирован почти/первоначально в то же время. Это показывает пример «обратного» влияния репутации человека на научное мышление той эпохи.

Недавние эксперименты Лакура-Отта являются прорывом и демонстрируют «локально детерминированную модель квантовых измерений, основанную на детекторе». Это поразительное открытие, которое еще не получило широкого распространения. Это доказывает, что полный квантово-механический формализм может возникнуть при анализе чисто классических систем. Таким образом, это ставит под сомнение почти столетние утверждения о том, что квантовая механика по своей сути отличается от классической механики, которая теперь рассматривается не просто как система убеждений, а фактически как догма в этой области. Есть много других недавних разработок, которые оставляют щели/вмятины в этой длинной броне и, кажется, заставляют переоценивать/пересматривать [7] (но это, безусловно, будет длительный процесс, и это только начало).

Новые теории сравниваются с бомовской механикой, но они имеют совершенно другие и новые аспекты, и их не следует отвергать как опровержение. Один из наиболее полных обзоров на сегодняшний день был проведен Бушем.[5] это недавно поддерживается экспериментами! [6]

Так как же это возможно концептуально/теоретически? Одним из поразительных новых открытий является то, что вероятностный закон Борна в квантовой механике может возникать в классических системах. См., например, Qiaochu Yuan, «Конечная некоммутативная вероятность, правило Борна и коллапс волновой функции» [8] и другие гораздо более подробные анализы детекторов, взятые у Хренникова и «PCST», «Доквантовая классическая статистическая теория поля», что также известно как примерно полуклассические теории.

[9] Говорится о детекторе, отбрасывающем энергию, когда поступающая энергия не соответствует порогу энергии детектора (p9) и о детекторе, "съедающем часть энергии" (p10). Назовем это диссипативным детектором. другая похожая концепция измерения - это мертвое время детектора [10p5], когда «детектор не может взаимодействовать с входящим импульсом».

Похоже, что эти концепции похожи на очень сложное / всестороннее исследование теоремы Белла, в котором сигнальная система может иметь так называемые события «отмены», и которое обнаруживает, что более строгие версии неравенств Белла не нарушаются текущими экспериментами. ]

Они аналогичны «лазейке выборки» [12], которая не обязательно совпадает с так называемой лазейкой эффективности, потому что первая может все еще существовать, даже если эффективность детектора измеряется на уровне 100%!

Давайте более внимательно изучим концепцию диссипативного детектора и то, как он будет выглядеть теоретически. Рассмотрим следующий эскиз. Сферический одиночный волновой фронт распространяется в пространстве. Теперь представьте, что он проходит через детектор. детектор может находиться в области мертвого времени и не будет обнаруживать волновой фронт. Или он может обнаружить это. Это вероятностная природа света. Похоже, что возможное мертвое время не может быть сведено к нулю как физический закон, связанный/аналогичный принципу неопределенности Гейзенберга.

Другими словами, идеальных детекторов не существует. Единственные детекторы, которые у нас есть, сделаны из атомов, то есть частиц. Тогда тайна фотона окончательно разгадана. Фотон — это (вероятностное) взаимодействие между волновым фронтом и измерительным устройством, а именно атомом или другой частицей. На взаимодействие можно ссылаться только апостериорно, а не априори . Другими словами, даже детектор, состоящий из одного атома, будет обладать этим свойством «мертвого времени» и рассеивания энергии. Итак, у нас также есть некоторая интерпретация так называемых виртуальных частиц .

Другие могут подвергнуть сомнению «сферический одиночный волновой фронт», путешествующий в пространстве. Именно эта картина теперь поддерживается новой моделью пространства-времени, подробно описанной Теневым/Хорстемейером, «ткань пространства-времени».[14] они, кажется, не уделяют особого внимания тензору электромагнитных напряжений, но одно очевидное обобщение их работы состоит в том, что электромагнитные волны являются s-волнами в ткани пространства-времени.

Довольно простым экспериментом, демонстрирующим эти идеи, является эффект HBT. Представьте себе линию детекторов, находящихся на одинаковом расстоянии от «однофотонного» источника, как простой способ повысить чувствительность обнаружения волнового фронта. Идея «однофотонного» источника может быть лучше визуализирована как источник «одноволнового фронта». Когда волновой фронт проходит через детекторы, каждый детектор может щелкнуть или не щелкнуть. Если какой-либо щелчок, был волновой фронт. Если ни один из них не щелкнет, возможно, фронт волны прошел, но все они, возможно, находились в своем «неотзывчивом» мертвом периоде. Общий комбинированный массив будет обнаруживать волновой фронт с большей точностью.

Этот эффект уже наблюдается, но не интерпретируется с этой точки зрения. В литературе это называется (анти) группированием фотонов . Многие другие эффекты в настоящее время неверно интерпретируются из-за тумана/дымки нашей в настоящее время туманной теории. Переделывать все это придется долго. Но такие переделки не являются чем-то неслыханным в истории науки, хотя они, как правило, касаются событий, происходящих раз в столетие, и буквально приводят к необходимости переписывать, например, учебники (но не все сразу!).[17] Их нельзя точно приурочить (по аналогии с землетрясениями ) и даже трудно распознать в середине, но некоторые признаки (собранные ссылки, например, также [18], многие другие, не цитируемые из-за ограничений места/формата, следующая цитата и т. д.) в настоящее время присутствуют. и мы, кажется, просрочены для одного.

«Я бы хотел, чтобы люди, разрабатывавшие квантовую механику в начале прошлого века, имели доступ к этим экспериментам, — сказал Милевски, — потому что тогда вся история квантовой механики могла бы быть другой».[7]

[1] слепые и слон / википедия

[2] Корпускулярная теория света / википедия

[3] Локально детерминированная модель квантовых измерений на основе детекторов / La Cour

[4] Квантовый компьютер, эмулированный классической системой / physorg

[5] Пилот-волновая гидродинамика / Буш

[6] новая поддержка альтернативного квантового представления /Wolchover

[7] Все это время мы неправильно интерпретировали квантовую механику? / Волховер

[8] Конечная некоммутативная вероятность, правило Борна и коллапс волновой функции Цяочу Юань

[9] Правило Борна по измерениям классических сигналов правильно откалиброванными пороговыми детекторами / Хренников .

[10] Доквантовая классическая статистическая теория поля: моделирование вероятностей обнаружения фотонов с помощью классического броуновского движения / Хренников

[11] Надежные неравенства Белла от коммуникационной сложности / Ла Плант.

[12] Лазейки в тестовых экспериментах Белла / wikipedia

[13] Нарушение неравенства Белла без лазеек с использованием электронных спинов, разделенных на 1,3 километра / Хенсен и др .

[14] Механика пространства-времени — взгляд механики твердого тела на общую теорию относительности / Тенев, Хорстемейер

[15] Тензор энергии электромагнитного напряжения / википедия

[16] Эффект HBT / википедия

[17] Смена парадигмы / Википедия

[18] EmQM13: Конференция / материалы конференции Emergent Quantum Mechanics 2013.

Самый простой способ понять фотон состоит в том, что это частица, которая подчиняется квантовой механике, а не классической механике. В квантовой механике частицы не имеют положения, но мы можем рассчитать вероятность того, где частица будет найдена (или, в случае с фотоном, где она будет уничтожена). Как выразился Дирак

«В общем случае мы не можем говорить о том, что наблюдаемая имеет значение для конкретного состояния, но мы можем… говорить о вероятности того, что она имеет определенное значение для данного состояния, имея в виду вероятность того, что это заданное значение будет получено при выполнении измерение наблюдаемого».

Расчет квантовых вероятностей неизбежно отличается от расчета классических вероятностей, потому что квантовая механика описывает действительно неопределенные процессы, тогда как в стандартной теории вероятностей результаты определяются неизвестными. По глубоким и тонким причинам, связанным с математическими основами квантовой механики (обычно не освещаемыми в стандартных учебниках), интерпретация вероятности требует, чтобы расчет следовал законам волновой механики, создавая иллюзию того, что частицы обладают волновыми свойствами.

Я показал математический аргумент в «Математике гравитации и квантов».

Уместные цитаты Эйнштейна :

  • Все пятьдесят лет сознательных размышлений не приблизили меня к ответу на вопрос: « Что такое кванты света? Конечно, сегодня каждый негодяй думает, что знает ответ, но он заблуждается.

  • Всю оставшуюся жизнь я буду размышлять о том, что такое свет .

Есть ли в фотоне что-то большее, чем можно объяснить только изменениями свойств его излучателя и поглотителя? (Ответ повторен 6-11-16)

После чуть большего количества отрицательных, чем положительных отзывов, а также полезной поправки, в общем, пора перестать копать себя в яму.

Тем не менее, просматривая свежую литературу, позвольте мне в заключение обратить внимание критиков на недавнюю статью в Quantum Stud.: Math. Найденный. (2016) 3:147–160 (копия приложена Рашковский (2016) ), который также доказывает отсутствие фотона, но с помощью полуклассического подхода. Хотя он рассматривает этот вопрос гораздо глубже и подробнее, чем я мог бы достичь, я полагаю, что он согласуется с мнением о том, что, в конечном счете, характеристики фотона лучше всего искать в свойствах излучателя и поглотителя, а не приписывать их свойствам. вымышленный фотон… или вообще какая разница, считаем ли мы его «настоящим» или нет?

Поскольку комментарии показали, что часть моего аргумента ошибочна, ссылки на «точки совпадения пространства-времени» и «контактное взаимодействие Минковского-пространства» были удалены.

Тем не менее, ссылки в приведенном выше вкладе Артура Ноймайера включают примеры выдающихся физиков, утверждающих, что фотонов не существует:

  • П. У. Дэвис: «В своей статье с провокационным названием «Частицы не существуют» Пол Дэвис выявляет несколько серьезных трудностей, связанных с любой общепринятой концепцией фотона, связанной с частицами».

  • Говорят, что лауреат Нобелевской премии Уиллис Лэмб «утверждал, что фотонов не существует».

Я предлагаю элементарный аргумент, опираясь на лекции Фейнмана (том 1, конец раздела 17-2 на странице 17-4):

Таким образом, на нашей диаграмме пространства-времени мы имели бы примерно следующее представление: под углом 45° проходят две линии (на самом деле, в четырех измерениях это будут «конусы», называемые световыми конусами), и точки на этих линиях равны все на нулевом расстоянии от начала координат. Там, где свет выходит из данной точки, всегда отделяется от нее нулевым интервалом, как это видно из уравнения (17.5). Между прочим, мы только что доказали, что если свет движется со скоростью с за один системе, то в другой он движется со скоростью с , ибо если интервал один и тот же в обеих системах, т. е. ноль в одной и ноль в другой, то утверждать, что скорость распространения света неизменна, — это то же самое, что говорить, что интервал равен нулю».

в котором он утверждает, что точки на световом конусе находятся на нулевом расстоянии от начала координат и друг от друга. Конечно, он имеет в виду нулевое 4-расстояние в пространстве-времени из-за того, что метрика Минковского требует вычитания между 3-пространственным интервалом и c, умноженным на временной интервал.

Таким образом, можно утверждать, что фотоны не имеют независимого существования в пространстве-времени: они «появляются» только как потеря, испуская из источника, и приобретая при поглощении [остальная часть первоначального предложения удалена].

С этой точки зрения фотоны в некотором смысле (как упоминали другие в более изощренных ответах) просто удобные вымыслы математики, которые объясняют, что происходит при наблюдаемой передаче энергии между излучателем и поглотителем через трехмерное пространство и временной промежуток, который мы соединяем, используя математику, описывающую волновые движения. Это говорит о том, что характеристики фотона (например, спин) следует искать в свойствах излучателя и поглотителя , а не приписывать фиктивному фотону…

Отвечая на голосующих против (NB, было и несколько голосовавших за). Не могли бы вы указать, где именно идея падает. Я просто озадачен тем, что при относительном невежестве моего химика кажется явным следствием СТО, что прохождение того, что мы описываем как фотон, есть только взаимодействие излучателя и поглотителя, которые, согласно Фейнману, находятся в прямом «4». -контакт» – их не разделяет конечный пространственно-временной интервал. Мой вопрос является попыткой получить разъяснения по этой кажущейся загадке. Спасибо
Нулевое пространственно-временное расстояние — это не то же самое, что «совпадающие точки», «метрика» Минковского принципиально не индуцирует правильную метрику в математическом смысле, согласно которой две точки с нулевым расстоянием одинаковы, поэтому часть о «контактном взаимодействии» просто не соответствует действительности. -чувственный. Существует бесконечно много различных точек, которые находятся на нулевом пространственно-временном интервале друг от друга, и это не транзитивное отношение (рассмотрите пересечение двух световых конусов пространственно разделенных точек - пересечение находится на нулевом расстоянии от обеих, но две точки нет).
@igael Спасибо за редактирование - улучшает презентацию
@ACuriousMind - Спасибо. Где я могу найти (или какое ключевое слово) что-нибудь о разнице между «метрикой» Минковского и тем, что вы называете правильной метрикой? У меня было другое сообщение, которое разубедило меня в понятии контактного взаимодействия, поэтому я ценю подтверждение (при условии, что вы тоже не «UM»)
То, что я называю «правильной» метрикой, было бы метрикой в ​​строгом математическом смысле . Это в точности соответствует нашей интуиции о расстоянии, в частности тому факту, что две точки с нулевым метрическим расстоянием являются одной и той же точкой, т. е. «соприкасаются». Понятие «метрики» в SR/GR — это понятие метрического тензора , которое дает «правильную» метрику только в римановом случае, но SR/GR является только лоренцевым/псевдоримановым.
@ACuriousMind Итак, корень моей ошибки заключается в том, что я ошибочно принял свойства метрики Минковского за свойства строго математической, по крайней мере, в отношении «контактного взаимодействия». Я предполагаю, что проблема заключается в изменении знака в подписи. Несмотря на это, мне все еще нужно убедить себя, что фотон нужен нам не только как удобная фикция, связывающая излучатель и поглотитель. Еще раз спасибо.
@ user40110 Помимо SR, что заставляет вас верить, что фотонов может вообще не существовать? Уравнение Е знак равно час ф или же Е знак равно 2.210 Икс 10 42 с ф заставить вас поверить в это? Какая часть этого уравнения заставит кого-то поверить в существование частицы?
@Inquisitive Ответ, который следует следовать, когда мне посоветовали опубликовать файл (мне некуда его опубликовать со ссылкой), но я думаю, что это слишком долго для публикации в виде открытого комментария (~ 2500 слов).
@ user40110 Я буду с нетерпением ждать твоего поста. Или вы можете просто урезать его, сохранив основные моменты.
@Inquisitive Не имея возможности написать адекватный ответ на ваш 1-й вопрос в отведенном месте, я разместил его в виде PDF-файла в начале темы для соответствующего вопроса на ResearchGate по адресу researchgate.net/post/… , который я начал в чтобы получить больше обсуждения, прежде чем другие комментируют здесь. Файл фокусируется на вашем 1-м вопросе: «Кроме СТО, что заставляет вас верить, что фотонов может вообще не существовать?» в качестве обзора моей позиции и предыдущих аргументов. Мои ответы на ваши 2-й и 3-й вопросы: Нет и Вероятно_нет .
@ Любознательный Итак, вы смогли увидеть ответ, который, как вы сказали, будете «с нетерпением ждать»?
25-5-18 Я был бы признателен за возможность извлечь уроки из сегодняшнего отрицательного голосования...